Answer (C) is correct . The approach to the solution is to weight (multiply) the probabilities for each level of demand by the payoff for that level of demand. Demand Payoff Probability Weighted Payoffs 0 $(160) ¡Á .1 = $(16) 2 (40) ¡Á .3 = (12) 4 80 ¡Á .4 = 32 6 80 ¡Á .2 = 16 ? ? Expected Profit $ 20 Answer (A) is incorrect because This figure is the payoff amount when demand is 2 units. Answer (B) is incorrect because This figure is the payoff amount when demand is 4 or 6 units. Answer (D) is incorrect because This figure is calculated by adding the amounts in the column for a demand of 4 units.
|